You are on page 1of 37

Definitions

IAS 39 incorporates the definitions of the following items from IAS 32 Financial Instruments: Presentation:
[IAS 39.8]
 financial instrument
 financial asset
 financial liability
 equity instrument.
Note: Where an entity applies IFRS 9 Financial Instruments prior to its mandatory application date (1
January 2015), definitions of the following terms are also incorporated from IFRS 9: derecognition,
derivative, fair value, financial guarantee contract. The definition of those terms outlined below (as
relevant) are those from IAS 39.

These questions help you recognize your existing background Yes No


knowledge on the topic. Answer honestly.
Does the standard definition given above match your personal
definition for investments?
Can you differentiate the different types of investments?
Have you read the standard for measuring investments?
Can you determine when to derecognize investments from the
books?
Have you read the standard for presentation and disclosure of
investments?
Total (Raw Score)

IAS 39 Financial Instruments: Recognition and Measurement by Silvia of CPDbox.


View at: https://www.youtube.com/watch?v=qDb0rbQWoFw

IAS 39 — Financial Instruments: Recognition and Measurement


Scope
Scope exclusions
IAS 39 applies to all types of financial instruments except for the following, which are scoped out of IAS 39:
[IAS 39.2]
 interests in subsidiaries, associates, and joint ventures accounted for under IAS 27 Consolidated and
Separate Financial Statements, IAS 28 Investments in Associates, or IAS 31 Interests in Joint
Ventures (or, for periods beginning on or after 1 January 2013, IFRS 10 Consolidated Financial
Statements, IAS 27 Separate Financial Statements or IAS 28 Investments in Associates and Joint
Ventures); however IAS 39 applies in cases where under those standards such interests are to be
accounted for under IAS 39. The standard also applies to most derivatives on an interest in a subsidiary,
associate, or joint venture
 employers' rights and obligations under employee benefit plans to which IAS 19 Employee
Benefits applies
 forward contracts between an acquirer and selling shareholder to buy or sell an acquiree that will result
in a business combination at a future acquisition date
 rights and obligations under insurance contracts, except IAS 39 does apply to financial instruments
that take the form of an insurance (or reinsurance) contract but that principally involve the transfer of
financial risks and derivatives embedded in insurance contracts
 financial instruments that meet the definition of own equity under IAS 32 Financial Instruments:
Presentation
 financial instruments, contracts and obligations under share-based payment transactions to
which IFRS 2 Share-based Payment applies
 rights to reimbursement payments to which IAS 37 Provisions, Contingent Liabilities and Contingent
Assets applies
Leases
IAS 39 applies to lease receivables and payables only in limited respects: [IAS 39.2(b)]
 IAS 39 applies to lease receivables with respect to the derecognition and impairment provisions
 IAS 39 applies to lease payables with respect to the derecognition provisions
 IAS 39 applies to derivatives embedded in leases.
Financial guarantees
IAS 39 applies to financial guarantee contracts issued. However, if an issuer of financial guarantee contracts
has previously asserted explicitly that it regards such contracts as insurance contracts and has used accounting
applicable to insurance contracts, the issuer may elect to apply either IAS 39 or IFRS 4 Insurance Contracts to
such financial guarantee contracts. The issuer may make that election contract by contract, but the election for
each contract is irrevocable.
Accounting by the holder is excluded from the scope of IAS 39 and IFRS 4 (unless the contract is a reinsurance
contract). Therefore, paragraphs 10-12 of IAS 8 Accounting Policies, Changes in Accounting Estimates and
Errors apply. Those paragraphs specify criteria to use in developing an accounting policy if no IFRS applies
specifically to an item.
Loan commitments
Loan commitments are outside the scope of IAS 39 if they cannot be settled net in cash or another financial
instrument, they are not designated as financial liabilities at fair value through profit or loss, and the entity
does not have a past practice of selling the loans that resulted from the commitment shortly after origination.
An issuer of a commitment to provide a loan at a below-market interest rate is required initially to recognise
the commitment at its fair value; subsequently, the issuer will remeasure it at the higher of (a) the amount
recognised under IAS 37 and (b) the amount initially recognised less, where appropriate, cumulative
amortisation recognised in accordance with IAS 18. An issuer of loan commitments must apply IAS 37 to
other loan commitments that are not within the scope of IAS 39 (that is, those made at market or above). Loan
commitments are subject to the derecognition provisions of IAS 39. [IAS 39.4]
Contracts to buy or sell financial items
Contracts to buy or sell financial items are always within the scope of IAS 39 (unless one of the other
exceptions applies).
Contracts to buy or sell non-financial items
Contracts to buy or sell non-financial items are within the scope of IAS 39 if they can be settled net in cash or
another financial asset and are not entered into and held for the purpose of the receipt or delivery of a non-
financial item in accordance with the entity's expected purchase, sale, or usage requirements. Contracts to buy
or sell non-financial items are inside the scope if net settlement occurs. The following situations constitute net
settlement: [IAS 39.5-6]
 the terms of the contract permit either counterparty to settle net
 there is a past practice of net settling similar contracts
 there is a past practice, for similar contracts, of taking delivery of the underlying and selling it within
a short period after delivery to generate a profit from short-term fluctuations in price, or from a dealer's
margin, or
 the non-financial item is readily convertible to cash
Weather derivatives
Although contracts requiring payment based on climatic, geological, or other physical variable were generally
excluded from the original version of IAS 39, they were added to the scope of the revised IAS 39 in December
2003 if they are not in the scope of IFRS 4. [IAS 39.AG1]
A derivative is a financial instrument:
 Whose value changes in response to the change in an underlying variable such as an interest rate,
commodity or security price, or index;

 That requires no initial investment, or one that is smaller than would be required for a contract with
similar response to changes in market factors; and

 That is settled at a future date. [IAS 39.9]


Examples of derivatives

Forwards: Contracts to purchase or sell a specific quantity of a financial instrument, a


commodity, or a foreign currency at a specified price determined at the outset, with delivery
or settlement at a specified future date. Settlement is at maturity by actual delivery of the
item specified in the contract, or by a net cash settlement.
Interest rate swaps and forward rate agreements: Contracts to exchange cash flows as
of a specified date or a series of specified dates based on a notional amount and fixed and
floating rates.
Futures: Contracts similar to forwards but with the following differences: futures are
generic exchange-traded, whereas forwards are individually tailored. Futures are generally
settled through an offsetting (reversing) trade, whereas forwards are generally settled by
delivery of the underlying item or cash settlement.
Options: Contracts that give the purchaser the right, but not the obligation, to buy (call
option) or sell (put option) a specified quantity of a particular financial instrument,
commodity, or foreign currency, at a specified price (strike price), during or at a specified
period of time. These can be individually written or exchange-traded. The purchaser of the
option pays the seller (writer) of the option a fee (premium) to compensate the seller for the
risk of payments under the option.
Caps and floors: These are contracts sometimes referred to as interest rate options. An
interest rate cap will compensate the purchaser of the cap if interest rates rise above a
predetermined rate (strike rate) while an interest rate floor will compensate the purchaser if
rates fall below a predetermined rate.
Embedded derivatives
Some contracts that themselves are not financial instruments may nonetheless have financial instruments
embedded in them. For example, a contract to purchase a commodity at a fixed price for delivery at a future
date has embedded in it a derivative that is indexed to the price of the commodity.
An embedded derivative is a feature within a contract, such that the cash flows associated with that feature
behave in a similar fashion to a stand-alone derivative. In the same way that derivatives must be accounted
for at fair value on the balance sheet with changes recognised in the income statement, so must some embedded
derivatives. IAS 39 requires that an embedded derivative be separated from its host contract and accounted
for as a derivative when: [IAS 39.11]
 the economic risks and characteristics of the embedded derivative are not closely related to those of
the host contract
 a separate instrument with the same terms as the embedded derivative would meet the definition of a
derivative, and
 the entire instrument is not measured at fair value with changes in fair value recognised in the income
statement
If an embedded derivative is separated, the host contract is accounted for under the appropriate standard (for
instance, under IAS 39 if the host is a financial instrument). Appendix A to IAS 39 provides examples of
embedded derivatives that are closely related to their hosts, and of those that are not.
Examples of embedded derivatives that are not closely related to their hosts (and therefore must be separately
accounted for) include:
 the equity conversion option in debt convertible to ordinary shares (from the perspective of the holder
only) [IAS 39.AG30(f)]
 commodity indexed interest or principal payments in host debt contracts[IAS 39.AG30(e)]
 cap and floor options in host debt contracts that are in-the-money when the instrument was issued
[IAS 39.AG33(b)]
 leveraged inflation adjustments to lease payments [IAS 39.AG33(f)]
 currency derivatives in purchase or sale contracts for non-financial items where the foreign currency
is not that of either counterparty to the contract, is not the currency in which the related good or service
is routinely denominated in commercial transactions around the world, and is not the currency that is
commonly used in such contracts in the economic environment in which the transaction takes place.
[IAS 39.AG33(d)]
If IAS 39 requires that an embedded derivative be separated from its host contract, but the entity is unable to
measure the embedded derivative separately, the entire combined contract must be designated as a financial
asset as at fair value through profit or loss). [IAS 39.12]
Classification as liability or equity
Since IAS 39 does not address accounting for equity instruments issued by the reporting enterprise but it does
deal with accounting for financial liabilities, classification of an instrument as liability or as equity is
critical. IAS 32 Financial Instruments: Presentation addresses the classification question.
Classification of financial assets
IAS 39 requires financial assets to be classified in one of the following categories: [IAS 39.45]
 Financial assets at fair value through profit or loss
 Available-for-sale financial assets
 Loans and receivables
 Held-to-maturity investments
Those categories are used to determine how a particular financial asset is recognised and measured in the
financial statements.
Financial assets at fair value through profit or loss. This category has two subcategories:
 Designated. The first includes any financial asset that is designated on initial recognition as one to be
measured at fair value with fair value changes in profit or loss.
 Held for trading. The second category includes financial assets that are held for trading. All
derivatives (except those designated hedging instruments) and financial assets acquired or held for the
purpose of selling in the short term or for which there is a recent pattern of short-term profit taking are
held for trading. [IAS 39.9]
Available-for-sale financial assets (AFS) are any non-derivative financial assets designated on initial
recognition as available for sale or any other instruments that are not classified as as (a) loans and receivables,
(b) held-to-maturity investments or (c) financial assets at fair value through profit or loss. [IAS 39.9] AFS
assets are measured at fair value in the balance sheet. Fair value changes on AFS assets are recognised directly
in equity, through the statement of changes in equity, except for interest on AFS assets (which is recognised
in income on an effective yield basis), impairment losses and (for interest-bearing AFS debt instruments)
foreign exchange gains or losses. The cumulative gain or loss that was recognised in equity is recognised in
profit or loss when an available-for-sale financial asset is derecognised. [IAS 39.55(b)]
Loans and receivables are non-derivative financial assets with fixed or determinable payments that are not
quoted in an active market, other than held for trading or designated on initial recognition as assets at fair
value through profit or loss or as available-for-sale. Loans and receivables for which the holder may not
recover substantially all of its initial investment, other than because of credit deterioration, should be classified
as available-for-sale.[IAS 39.9] Loans and receivables are measured at amortised cost. [IAS 39.46(a)]
Held-to-maturity investments are non-derivative financial assets with fixed or determinable payments that
an entity intends and is able to hold to maturity and that do not meet the definition of loans and receivables
and are not designated on initial recognition as assets at fair value through profit or loss or as available for
sale. Held-to-maturity investments are measured at amortised cost. If an entity sells a held-to-maturity
investment other than in insignificant amounts or as a consequence of a non-recurring, isolated event beyond
its control that could not be reasonably anticipated, all of its other held-to-maturity investments must be
reclassified as available-for-sale for the current and next two financial reporting years. [IAS 39.9] Held-to-
maturity investments are measured at amortised cost. [IAS 39.46(b)]
Classification of financial liabilities
IAS 39 recognises two classes of financial liabilities: [IAS 39.47]
 Financial liabilities at fair value through profit or loss
 Other financial liabilities measured at amortised cost using the effective interest method
The category of financial liability at fair value through profit or loss has two subcategories:
 Designated. a financial liability that is designated by the entity as a liability at fair value through profit
or loss upon initial recognition
 Held for trading. a financial liability classified as held for trading, such as an obligation for securities
borrowed in a short sale, which have to be returned in the future
Initial recognition
IAS 39 requires recognition of a financial asset or a financial liability when, and only when, the entity becomes
a party to the contractual provisions of the instrument, subject to the following provisions in respect of regular
way purchases. [IAS 39.14]
Regular way purchases or sales of a financial asset. A regular way purchase or sale of financial assets is
recognised and derecognised using either trade date or settlement date accounting. [IAS 39.38] The method
used is to be applied consistently for all purchases and sales of financial assets that belong to the same category
of financial asset as defined in IAS 39 (note that for this purpose assets held for trading form a different
category from assets designated at fair value through profit or loss). The choice of method is an accounting
policy. [IAS 39.38]
IAS 39 requires that all financial assets and all financial liabilities be recognised on the balance sheet. That
includes all derivatives. Historically, in many parts of the world, derivatives have not been recognised on
company balance sheets. The argument has been that at the time the derivative contract was entered into, there
was no amount of cash or other assets paid. Zero cost justified non-recognition, notwithstanding that as time
passes and the value of the underlying variable (rate, price, or index) changes, the derivative has a positive
(asset) or negative (liability) value.
Initial measurement
Initially, financial assets and liabilities should be measured at fair value (including transaction costs, for assets
and liabilities not measured at fair value through profit or loss). [IAS 39.43]
Measurement subsequent to initial recognition
Subsequently, financial assets and liabilities (including derivatives) should be measured at fair value, with the
following exceptions: [IAS 39.46-47]
 Loans and receivables, held-to-maturity investments, and non-derivative financial liabilities should be
measured at amortised cost using the effective interest method.
 Investments in equity instruments with no reliable fair value measurement (and derivatives indexed to
such equity instruments) should be measured at cost.
 Financial assets and liabilities that are designated as a hedged item or hedging instrument are subject
to measurement under the hedge accounting requirements of the IAS 39.
 Financial liabilities that arise when a transfer of a financial asset does not qualify for derecognition, or
that are accounted for using the continuing-involvement method, are subject to particular measurement
requirements.
Fair value is the amount for which an asset could be exchanged, or a liability settled, between knowledgeable,
willing parties in an arm's length transaction. [IAS 39.9] IAS 39 provides a hierarchy to be used in determining
the fair value for a financial instrument: [IAS 39 Appendix A, paragraphs AG69-82]
 Quoted market prices in an active market are the best evidence of fair value and should be used, where
they exist, to measure the financial instrument.
 If a market for a financial instrument is not active, an entity establishes fair value by using a valuation
technique that makes maximum use of market inputs and includes recent arm's length market
transactions, reference to the current fair value of another instrument that is substantially the same,
discounted cash flow analysis, and option pricing models. An acceptable valuation technique
incorporates all factors that market participants would consider in setting a price and is consistent with
accepted economic methodologies for pricing financial instruments.
 If there is no active market for an equity instrument and the range of reasonable fair values is significant
and these estimates cannot be made reliably, then an entity must measure the equity instrument at cost
less impairment.
Amortised cost is calculated using the effective interest method. The effective interest rate is the rate that
exactly discounts estimated future cash payments or receipts through the expected life of the financial
instrument to the net carrying amount of the financial asset or liability. Financial assets that are not carried at
fair value though profit and loss are subject to an impairment test. If expected life cannot be determined
reliably, then the contractual life is used.
IAS 39 fair value option
IAS 39 permits entities to designate, at the time of acquisition or issuance, any financial asset or financial
liability to be measured at fair value, with value changes recognised in profit or loss. This option is available
even if the financial asset or financial liability would ordinarily, by its nature, be measured at amortised cost
– but only if fair value can be reliably measured.
In June 2005 the IASB issued its amendment to IAS 39 to restrict the use of the option to designate any
financial asset or any financial liability to be measured at fair value through profit and loss (the fair value
option). The revisions limit the use of the option to those financial instruments that meet certain conditions:
[IAS 39.9]
 the fair value option designation eliminates or significantly reduces an accounting mismatch, or
 a group of financial assets, financial liabilities or both is managed and its performance is evaluated on
a fair value basis by entity's management.
Once an instrument is put in the fair-value-through-profit-and-loss category, it cannot be reclassified out with
some exceptions. [IAS 39.50] In October 2008, the IASB issued amendments to IAS 39. The amendments
permit reclassification of some financial instruments out of the fair-value-through-profit-or-loss category
(FVTPL) and out of the available-for-sale category – for more detail see IAS 39.50(c). In the event of
reclassification, additional disclosures are required under IFRS 7 Financial Instruments: Disclosures. In
March 2009 the IASB clarified that reclassifications of financial assets under the October 2008 amendments
(see above): on reclassification of a financial asset out of the 'fair value through profit or loss' category, all
embedded derivatives have to be (re)assessed and, if necessary, separately accounted for in financial
statements.
IAS 39 available for sale option for loans and receivables
IAS 39 permits entities to designate, at the time of acquisition, any loan or receivable as available for sale, in
which case it is measured at fair value with changes in fair value recognised in equity.
Impairment
A financial asset or group of assets is impaired, and impairment losses are recognised, only if there is objective
evidence as a result of one or more events that occurred after the initial recognition of the asset. An entity is
required to assess at each balance sheet date whether there is any objective evidence of impairment. If any
such evidence exists, the entity is required to do a detailed impairment calculation to determine whether an
impairment loss should be recognised. [IAS 39.58] The amount of the loss is measured as the difference
between the asset's carrying amount and the present value of estimated cash flows discounted at the financial
asset's original effective interest rate. [IAS 39.63]
Assets that are individually assessed and for which no impairment exists are grouped with financial assets
with similar credit risk statistics and collectively assessed for impairment. [IAS 39.64]
If, in a subsequent period, the amount of the impairment loss relating to a financial asset carried at amortised
cost or a debt instrument carried as available-for-sale decreases due to an event occurring after the impairment
was originally recognised, the previously recognised impairment loss is reversed through profit or loss.
Impairments relating to investments in available-for-sale equity instruments are not reversed through profit or
loss. [IAS 39.65]
Financial guarantees
A financial guarantee contract is a contract that requires the issuer to make specified payments to reimburse
the holder for a loss it incurs because a specified debtor fails to make payment when due. [IAS 39.9]
Under IAS 39 as amended, financial guarantee contracts are recognised:
 initially at fair value. If the financial guarantee contract was issued in a stand-alone arm's length
transaction to an unrelated party, its fair value at inception is likely to equal the consideration received,
unless there is evidence to the contrary.
 subsequently at the higher of (i) the amount determined in accordance with IAS 37 Provisions,
Contingent Liabilities and Contingent Assets and (ii) the amount initially recognised less, when
appropriate, cumulative amortisation recognised in accordance with IAS 18 Revenue. (If specified
criteria are met, the issuer may use the fair value option in IAS 39. Furthermore, different requirements
continue to apply in the specialised context of a 'failed' derecognition transaction.)
Some credit-related guarantees do not, as a precondition for payment, require that the holder is exposed to,
and has incurred a loss on, the failure of the debtor to make payments on the guaranteed asset when due. An
example of such a guarantee is a credit derivative that requires payments in response to changes in a specified
credit rating or credit index. These are derivatives and they must be measured at fair value under IAS 39.
Derecognition of a financial asset
The basic premise for the derecognition model in IAS 39 is to determine whether the asset under consideration
for derecognition is: [IAS 39.16]
 an asset in its entirety or
 specifically identified cash flows from an asset or
 a fully proportionate share of the cash flows from an asset or
 a fully proportionate share of specifically identified cash flows from a financial asset
Once the asset under consideration for derecognition has been determined, an assessment is made as to
whether the asset has been transferred, and if so, whether the transfer of that asset is subsequently eligible for
derecognition.
An asset is transferred if either the entity has transferred the contractual rights to receive the cash flows, or the
entity has retained the contractual rights to receive the cash flows from the asset, but has assumed a contractual
obligation to pass those cash flows on under an arrangement that meets the following three conditions:
[IAS 39.17-19]
 the entity has no obligation to pay amounts to the eventual recipient unless it collects equivalent
amounts on the original asset
 the entity is prohibited from selling or pledging the original asset (other than as security to the eventual
recipient),
 the entity has an obligation to remit those cash flows without material delay
Once an entity has determined that the asset has been transferred, it then determines whether or not it has
transferred substantially all of the risks and rewards of ownership of the asset. If substantially all the risks and
rewards have been transferred, the asset is derecognised. If substantially all the risks and rewards have been
retained, derecognition of the asset is precluded. [IAS 39.20]
If the entity has neither retained nor transferred substantially all of the risks and rewards of the asset, then the
entity must assess whether it has relinquished control of the asset or not. If the entity does not control the asset
then derecognition is appropriate; however if the entity has retained control of the asset, then the entity
continues to recognise the asset to the extent to which it has a continuing involvement in the asset. [IAS 39.30]
These various derecognition steps are summarised in the decision tree in AG36.
Derecognition of a financial liability
A financial liability should be removed from the balance sheet when, and only when, it is extinguished, that
is, when the obligation specified in the contract is either discharged or cancelled or expires. [IAS 39.39] Where
there has been an exchange between an existing borrower and lender of debt instruments with substantially
different terms, or there has been a substantial modification of the terms of an existing financial liability, this
transaction is accounted for as an extinguishment of the original financial liability and the recognition of a
new financial liability. A gain or loss from extinguishment of the original financial liability is recognised in
profit or loss. [IAS 39.40-41]
Hedge accounting
IAS 39 permits hedge accounting under certain circumstances provided that the hedging relationship is:
[IAS 39.88]
 formally designated and documented, including the entity's risk management objective and strategy
for undertaking the hedge, identification of the hedging instrument, the hedged item, the nature of the
risk being hedged, and how the entity will assess the hedging instrument's effectiveness and
 expected to be highly effective in achieving offsetting changes in fair value or cash flows attributable
to the hedged risk as designated and documented, and effectiveness can be reliably measured and
 assessed on an ongoing basis and determined to have been highly effective
Hedging instruments
Hedging instrument is an instrument whose fair value or cash flows are expected to offset changes in the fair
value or cash flows of a designated hedged item. [IAS 39.9]
All derivative contracts with an external counterparty may be designated as hedging instruments except for
some written options. A non-derivative financial asset or liability may not be designated as a hedging
instrument except as a hedge of foreign currency risk. [IAS 39.72]
For hedge accounting purposes, only instruments that involve a party external to the reporting entity can be
designated as a hedging instrument. This applies to intragroup transactions as well (with the exception of
certain foreign currency hedges of forecast intragroup transactions – see below). However, they may qualify
for hedge accounting in individual financial statements. [IAS 39.73]
Hedged items
Hedged item is an item that exposes the entity to risk of changes in fair value or future cash flows and is
designated as being hedged. [IAS 39.9]
A hedged item can be: [IAS 39.78-82]
 a single recognised asset or liability, firm commitment, highly probable transaction or a net investment
in a foreign operation
 a group of assets, liabilities, firm commitments, highly probable forecast transactions or net
investments in foreign operations with similar risk characteristics
 a held-to-maturity investment for foreign currency or credit risk (but not for interest risk or prepayment
risk)
 a portion of the cash flows or fair value of a financial asset or financial liability or
 a non-financial item for foreign currency risk only for all risks of the entire item
 in a portfolio hedge of interest rate risk (Macro Hedge) only, a portion of the portfolio of financial
assets or financial liabilities that share the risk being hedged
In April 2005, the IASB amended IAS 39 to permit the foreign currency risk of a highly probable intragroup
forecast transaction to qualify as the hedged item in a cash flow hedge in consolidated financial statements –
provided that the transaction is denominated in a currency other than the functional currency of the entity
entering into that transaction and the foreign currency risk will affect consolidated financial statements.
[IAS 39.80]
In 30 July 2008, the IASB amended IAS 39 to clarify two hedge accounting issues:
 inflation in a financial hedged item
 a one-sided risk in a hedged item.
Effectiveness
IAS 39 requires hedge effectiveness to be assessed both prospectively and retrospectively. To qualify for
hedge accounting at the inception of a hedge and, at a minimum, at each reporting date, the changes in the fair
value or cash flows of the hedged item attributable to the hedged risk must be expected to be highly effective
in offsetting the changes in the fair value or cash flows of the hedging instrument on a prospective basis, and
on a retrospective basis where actual results are within a range of 80% to 125%.
All hedge ineffectiveness is recognised immediately in profit or loss (including ineffectiveness within the 80%
to 125% window).
Categories of hedges
A fair value hedge is a hedge of the exposure to changes in fair value of a recognised asset or liability or a
previously unrecognised firm commitment or an identified portion of such an asset, liability or firm
commitment, that is attributable to a particular risk and could affect profit or loss. [IAS 39.86(a)] The gain or
loss from the change in fair value of the hedging instrument is recognised immediately in profit or loss. At the
same time the carrying amount of the hedged item is adjusted for the corresponding gain or loss with respect
to the hedged risk, which is also recognised immediately in net profit or loss. [IAS 39.89]
A cash flow hedge is a hedge of the exposure to variability in cash flows that (i) is attributable to a particular
risk associated with a recognised asset or liability (such as all or some future interest payments on variable
rate debt) or a highly probable forecast transaction and (ii) could affect profit or loss. [IAS 39.86(b)] The
portion of the gain or loss on the hedging instrument that is determined to be an effective hedge is recognised
in other comprehensive income. [IAS 39.95]
If a hedge of a forecast transaction subsequently results in the recognition of a financial asset or a financial
liability, any gain or loss on the hedging instrument that was previously recognised directly in equity is
'recycled' into profit or loss in the same period(s) in which the financial asset or liability affects profit or loss.
[IAS 39.97]
If a hedge of a forecast transaction subsequently results in the recognition of a non-financial asset or non-
financial liability, then the entity has an accounting policy option that must be applied to all such hedges of
forecast transactions: [IAS 39.98]
 Same accounting as for recognition of a financial asset or financial liability – any gain or loss on the
hedging instrument that was previously recognised in other comprehensive income is 'recycled' into
profit or loss in the same period(s) in which the non-financial asset or liability affects profit or loss.
 'Basis adjustment' of the acquired non-financial asset or liability – the gain or loss on the hedging
instrument that was previously recognised in other comprehensive income is removed from equity and
is included in the initial cost or other carrying amount of the acquired non-financial asset or liability.
A hedge of a net investment in a foreign operation as defined in IAS 21 The Effects of Changes in Foreign
Exchange Rates is accounted for similarly to a cash flow hedge. [IAS 39.102]
A hedge of the foreign currency risk of a firm commitment may be accounted for as a fair value hedge or
as a cash flow hedge.
Discontinuation of hedge accounting
Hedge accounting must be discontinued prospectively if: [IAS 39.91 and 39.101]
 the hedging instrument expires or is sold, terminated, or exercised
 the hedge no longer meets the hedge accounting criteria – for example it is no longer effective
 for cash flow hedges the forecast transaction is no longer expected to occur, or
 the entity revokes the hedge designation
In June 2013, the IASB amended IAS 39 to make it clear that there is no need to discontinue hedge accounting
if a hedging derivative is novated, provided certain criteria are met. [IAS 39.91 and IAS 39.101]
For the purpose of measuring the carrying amount of the hedged item when fair value hedge accounting ceases,
a revised effective interest rate is calculated. [IAS 39.BC35A]
If hedge accounting ceases for a cash flow hedge relationship because the forecast transaction is no longer
expected to occur, gains and losses deferred in other comprehensive income must be taken to profit or loss
immediately. If the transaction is still expected to occur and the hedge relationship ceases, the amounts
accumulated in equity will be retained in equity until the hedged item affects profit or loss. [IAS 39.101(c)]
If a hedged financial instrument that is measured at amortised cost has been adjusted for the gain or loss
attributable to the hedged risk in a fair value hedge, this adjustment is amortised to profit or loss based on a
recalculated effective interest rate on this date such that the adjustment is fully amortised by the maturity of
the instrument. Amortisation may begin as soon as an adjustment exists and must begin no later than when
the hedged item ceases to be adjusted for changes in its fair value attributable to the risks being hedged.

13.1
The following data pertains to Gob Corporation’s investments in marketable securities:
Market Value
Cost 12/31/07 12/31/06
Trading P 150,000 P 155,000 P 100,000
Available-for-sale 150,000 130,000 126,000
1. What amount should Gob Corporation report as unrealized holding gain in its 2007 income statement?
2. What amount should Gob Corporation report as unrealized loss on marketable equity securities at
December 31, 2007, in accumulated other comprehensive income in stockholders’ equity?

13.2
Eve Co. has investments in shares of common stock of Adam Corp. Company, bought as follows:
2003 1,000 shares – P 140,000
2005 500 shares – P 90,000
The following transactions took place in 2007 with respect to these holdings:
 April 10 By proper resolution, there was a 3 for 1 stock split and Eve Co. Company received
3,000 shares in addition to her original holdings.
 July 10 Eve Co. Company received a P0.60 per share cash dividend and also rights to
subscribed to one share at P40 each for every five shares held. On this date, shares of stock of Adam
Corp. Company were selling ex-rights at P55 per share and rights were selling at P2 each.
 July 20 Eve Co. Company exercised all her rights by buying the new shares and paid P36,000.
 Nov. 15 Eve Co. sold 1,000 shares at P60 each, taken from those acquired in 2003, less broker’s
commission of P750.
3. The investment in stock at year-end is:
4. The investment in stock at year-end from the 2003 purchase is:
5. The investment in stock at year-end from the 2005 purchase is:
6. The gain on sale of investment at year-end is:
7. How many shares were purchased during the year?

13.3
On December 31, 2006, DreamBig Company reported as Available-for-sale securities:
Attitude Company, 5,000 shares of common stock (a 1% interest) P 125,000
IstheKEY Company, 10,000 shares of common stock (a 2% interest) 160,000
2Success Company, 25,000 shares of common stock (a 10% interest) 700,000
Marketable equity securities, at cost P 985,000
Less: Valuation allowance 50,000
Marketable equity securities, at market P 935,000
Additional information:
• On May, 2007, Attitude Company issued a 10% stock dividend when the market price of its stock was
P24 per share.
• On November 1, 2007, Attitude Company paid a cash dividend of P0.75 per share.
• On August 5, 2007, IstheKEY Company issued to all shareholders, stock rights on the basis of one
right per share. Market prices at date of issue were P13.50 per share (ex-right) of stock and P1.50 per rights.
DreamBig Company sold all rights on December 16, 2007 for net proceeds of P18,800.
• On July 1, 2007, DreamBig Company paid P1,520,000 for 50,000 additional shares of 2Success
Company’s common stock which represented a 20% investment in 2Success Company. The fair value of all
of the 2Success Company’s identifiable assets net of liabilities was equal to their carrying amount of
P6,350,000. As a result of this transaction, DreamBig Company owns 30% of 2Success Company and can
exercise significant influence over 2Success Company’s operating and financial policies.
• DreamBig Company’s initial 10% interest of 25,000 shares of 2Success Company’s common stock
was acquired on January 2, 2006 for P700,000. At that date, the net assets of 2Success Company totaled
P5,800,000 and the fair value of 2Success’s identifiable assets net of liabilities was equal to their carrying
amount.
• Market prices per share of the marketable equity securities which were all listed in the stock exchange,
were as follows: At December 31
2006 2007
Attitude Company – common P 22 P 23
IstheKEY Company – common 15 14
2Success Company – common 27 29
• 2Success Company reported net income and paid dividends of:
Year Ended Div. per Share
Year ended December 31. 2006 P350,000 none
Six months ended June 30, 2007 200,000 none
Six months ended December 31, 2007 370,000 P 1.30
(dividend was paid on 10/1/07
• There were no other intercompany transactions between DreamBig Company and 2Success Company
and there were no impairment of 2Success Company’s asset at year-end.
8. The investment in Attitude Company common stock at year-end is:
9. The investment in Isthekey Company common stock at year-end is:
10. The investment in 2Success Company common stock at year-end is:
11. The recovery of market decline to be reported in the income statement is:
12. Dividend income to be reported in the income statement is:
13. Gain on sale of stock rights is:

Determine whether these statements relate to you. Answer Yes No


honestly to check your progress.
I am able to define investments and related terms.
I can identify investments from other types of assets.
I can differentiate the types of investments.
I understand the measuring standard for investments.
I can determine when to derecognize investments from the books.
I can determine the presentation and disclosure requirements for
investments.
Total (Raw Score)
Definitions
IAS 39 incorporates the definitions of the following items from IAS 32 Financial Instruments: Presentation:
[IAS 39.8]
 financial instrument
 financial asset
 financial liability
 equity instrument.
Note: Where an entity applies IFRS 9 Financial Instruments prior to its mandatory application date (1
January 2015), definitions of the following terms are also incorporated from IFRS 9: derecognition,
derivative, fair value, financial guarantee contract. The definition of those terms outlined below (as
relevant) are those from IAS 39.

These questions help you recognize your existing background Yes No


knowledge on the topic. Answer honestly.
Does the standard definition given above match your personal
definition for debt securities?
Can you differentiate debt securities from the other types of
investments?
Have you read the standard for measuring debt securities?
Can you determine when to derecognize debt securities from the
books?
Have you read the standard for presentation and disclosure of debt
securities?
Total (Raw Score)

IFRS 9 Financial Instruments - 2017 update by Silvia of CPDbox.


View at: https://www.youtube.com/watch?v=mWGctejmSZk

IFRS 9 — Financial Instruments


Debt instruments
A debt instrument that meets the following two conditions must be measured at amortised cost (net of any
write down for impairment) unless the asset is designated at FVTPL under the fair value option (see below):
[IFRS 9, paragraph 4.1.2]
 Business model test: The objective of the entity's business model is to hold the financial asset to
collect the contractual cash flows (rather than to sell the instrument prior to its contractual maturity to
realise its fair value changes).
 Cash flow characteristics test: The contractual terms of the financial asset give rise on specified dates
to cash flows that are solely payments of principal and interest on the principal amount outstanding.
Assessing the cash flow characteristics also includes an analysis of changes in the timing or in the amount of
payments. It is necessary to assess whether the cash flows before and after the change represent only
repayments of the nominal amount and an interest rate based on them.
The right of termination may for example be in accordance with the cash flow condition if, in the case of
termination, the only outstanding payments consist of principal and interest on the principal amount and an
appropriate compensation payment where applicable. In October 2017, the IASB clarified that the
compensation payments can also have a negative sign.*
*Prepayment Features with Negative Compensation (Amendments to IFRS 9); to be applied retrospectively
for fiscal years beginning on or after 1 January 2019; early application permitted
A debt instrument that meets the following two conditions must be measured at FVTOCI unless the asset is
designated at FVTPL under the fair value option (see below):
[IFRS 9, paragraph 4.1.2A]
 Business model test: The financial asset is held within a business model whose objective is achieved
by both collecting contractual cash flows and selling financial assets.
 Cash flow characteristics test: The contractual terms of the financial asset give rise on specified dates
to cash flows that are solely payments of principal and interest on the principal amount outstanding.
All other debt instruments must be measured at fair value through profit or loss (FVTPL). [IFRS 9, paragraph
4.1.4]
Fair value option
Even if an instrument meets the two requirements to be measured at amortised cost or FVTOCI, IFRS 9
contains an option to designate, at initial recognition, a financial asset as measured at FVTPL if doing so
eliminates or significantly reduces a measurement or recognition inconsistency (sometimes referred to as an
'accounting mismatch') that would otherwise arise from measuring assets or liabilities or recognising the gains
and losses on them on different bases. [IFRS 9, paragraph 4.1.5]
Equity instruments
All equity investments in scope of IFRS 9 are to be measured at fair value in the statement of financial position,
with value changes recognised in profit or loss, except for those equity investments for which the entity has
elected to present value changes in 'other comprehensive income'. There is no 'cost exception' for unquoted
equities.
'Other comprehensive income' option
If an equity investment is not held for trading, an entity can make an irrevocable election at initial recognition
to measure it at FVTOCI with only dividend income recognised in profit or loss. [IFRS 9, paragraph 5.7.5]
Measurement guidance
Despite the fair value requirement for all equity investments, IFRS 9 contains guidance on when cost may be
the best estimate of fair value and also when it might not be representative of fair value.
Subsequent measurement of financial liabilities
IFRS 9 doesn't change the basic accounting model for financial liabilities under IAS 39. Two measurement
categories continue to exist: FVTPL and amortised cost. Financial liabilities held for trading are measured at
FVTPL, and all other financial liabilities are measured at amortised cost unless the fair value option is applied.
[IFRS 9, paragraph 4.2.1]
Fair value option
IFRS 9 contains an option to designate a financial liability as measured at FVTPL if [IFRS 9, paragraph 4.2.2]:
 doing so eliminates or significantly reduces a measurement or recognition inconsistency (sometimes
referred to as an 'accounting mismatch') that would otherwise arise from measuring assets or liabilities
or recognising the gains and losses on them on different bases, or
 the liability is part or a group of financial liabilities or financial assets and financial liabilities that is
managed and its performance is evaluated on a fair value basis, in accordance with a documented risk
management or investment strategy, and information about the group is provided internally on that
basis to the entity's key management personnel.
A financial liability which does not meet any of these criteria may still be designated as measured at FVTPL
when it contains one or more embedded derivatives that sufficiently modify the cash flows of the liability and
are not clearly closely related. [IFRS 9, paragraph 4.3.5]
IFRS 9 requires gains and losses on financial liabilities designated as at FVTPL to be split into the amount of
change in fair value attributable to changes in credit risk of the liability, presented in other comprehensive
income, and the remaining amount presented in profit or loss. The new guidance allows the recognition of the
full amount of change in the fair value in profit or loss only if the presentation of changes in the liability's
credit risk in other comprehensive income would create or enlarge an accounting mismatch in profit or loss.
That determination is made at initial recognition and is not reassessed. [IFRS 9, paragraphs 5.7.7-5.7.8]
Amounts presented in other comprehensive income shall not be subsequently transferred to profit or loss, the
entity may only transfer the cumulative gain or loss within equity.
Derecognition of financial assets
The basic premise for the derecognition model in IFRS 9 (carried over from IAS 39) is to determine whether
the asset under consideration for derecognition is: [IFRS 9, paragraph 3.2.2]
 an asset in its entirety or
 specifically identified cash flows from an asset (or a group of similar financial assets) or
 a fully proportionate (pro rata) share of the cash flows from an asset (or a group of similar financial
assets). or
 a fully proportionate (pro rata) share of specifically identified cash flows from a financial asset (or a
group of similar financial assets)
Once the asset under consideration for derecognition has been determined, an assessment is made as to
whether the asset has been transferred, and if so, whether the transfer of that asset is subsequently eligible for
derecognition.
An asset is transferred if either the entity has transferred the contractual rights to receive the cash flows, or the
entity has retained the contractual rights to receive the cash flows from the asset, but has assumed a contractual
obligation to pass those cash flows on under an arrangement that meets the following three conditions: [IFRS
9, paragraphs 3.2.4-3.2.5]
 the entity has no obligation to pay amounts to the eventual recipient unless it collects equivalent
amounts on the original asset
 the entity is prohibited from selling or pledging the original asset (other than as security to the eventual
recipient),
 the entity has an obligation to remit those cash flows without material delay
Once an entity has determined that the asset has been transferred, it then determines whether or not it has
transferred substantially all of the risks and rewards of ownership of the asset. If substantially all the risks and
rewards have been transferred, the asset is derecognised. If substantially all the risks and rewards have been
retained, derecognition of the asset is precluded. [IFRS 9, paragraphs 3.2.6(a)-(b)]
If the entity has neither retained nor transferred substantially all of the risks and rewards of the asset, then the
entity must assess whether it has relinquished control of the asset or not. If the entity does not control the asset
then derecognition is appropriate; however if the entity has retained control of the asset, then the entity
continues to recognise the asset to the extent to which it has a continuing involvement in the asset. [IFRS 9,
paragraph 3.2.6(c)]
These various derecognition steps are summarised in the decision tree in paragraph B3.2.1.
Derecognition of financial liabilities
A financial liability should be removed from the balance sheet when, and only when, it is extinguished, that
is, when the obligation specified in the contract is either discharged or cancelled or expires. [IFRS 9, paragraph
3.3.1] Where there has been an exchange between an existing borrower and lender of debt instruments with
substantially different terms, or there has been a substantial modification of the terms of an existing financial
liability, this transaction is accounted for as an extinguishment of the original financial liability and the
recognition of a new financial liability. A gain or loss from extinguishment of the original financial liability
is recognised in profit or loss. [IFRS 9, paragraphs 3.3.2-3.3.3]
Impairment
The impairment model in IFRS 9 is based on the premise of providing for expected losses.
Scope
IFRS 9 requires that the same impairment model apply to all of the following:
[IFRS 9 paragraph 5.5.1]
 Financial assets measured at amortised cost;
 Financial assets mandatorily measured at FVTOCI;
 Loan commitments when there is a present obligation to extend credit (except where these are
measured at FVTPL);
o Financial guarantee contracts to which IFRS 9 is applied (except those measured at FVTPL);
o Lease receivables within the scope of IAS 17 Leases; and
o Contract assets within the scope of IFRS 15 Revenue from Contracts with Customers (i.e. rights
to consideration following transfer of goods or services).
General approach
With the exception of purchased or originated credit impaired financial assets (see below), expected credit
losses are required to be measured through a loss allowance at an amount equal to:
[IFRS 9 paragraphs 5.5.3 and 5.5.5]
 the 12-month expected credit losses (expected credit losses that result from those default events on the
financial instrument that are possible within 12 months after the reporting date); or
 full lifetime expected credit losses (expected credit losses that result from all possible default events
over the life of the financial instrument).
A loss allowance for full lifetime expected credit losses is required for a financial instrument if the credit risk
of that financial instrument has increased significantly since initial recognition, as well as to contract assets or
trade receivables that do not constitute a financing transaction in accordance with IFRS 15. [IFRS 9 paragraphs
5.5.3 and 5.5.15]
Additionally, entities can elect an accounting policy to recognise full lifetime expected losses for all contract
assets and/or all trade receivables that do constitute a financing transaction in accordance with IFRS 15. The
same election is also separately permitted for lease receivables. [IFRS 9 paragraph 5.5.16]
For all other financial instruments, expected credit losses are measured at an amount equal to the 12-month
expected credit losses. [IFRS 9 paragraph 5.5.5]
Significant increase in credit risk
With the exception of purchased or originated credit-impaired financial assets (see below), the loss allowance
for financial instruments is measured at an amount equal to lifetime expected losses if the credit risk of a
financial instrument has increased significantly since initial recognition, unless the credit risk of the financial
instrument is low at the reporting date in which case it can be assumed that credit risk on the financial
instrument has not increased significantly since initial recognition. [IFRS 9 paragraphs 5.5.3 and 5.5.10]
The Standard considers credit risk low if there is a low risk of default, the borrower has a strong capacity to
meet its contractual cash flow obligations in the near term and adverse changes in economic and business
conditions in the longer term may, but will not necessarily, reduce the ability of the borrower to fulfil its
contractual cash flow obligations. The Standard suggests that ‘investment grade’ rating might be an indicator
for a low credit risk. [IFRS 9 paragraphs B5.5.22 – B5.5.24]
The assessment of whether there has been a significant increase in credit risk is based on an increase in the
probability of a default occurring since initial recognition. Under the Standard, an entity may use various
approaches to assess whether credit risk has increased significantly (provided that the approach is consistent
with the requirements). An approach can be consistent with the requirements even if it does not include an
explicit probability of default occurring as an input. The application guidance provides a list of factors that
may assist an entity in making the assessment. Also, whilst in principle the assessment of whether a loss
allowance should be based on lifetime expected credit losses is to be made on an individual basis, some factors
or indicators might not be available at an instrument level. In this case, the entity should perform the
assessment on appropriate groups or portions of a portfolio of financial instruments.
The requirements also contain a rebuttable presumption that the credit risk has increased significantly when
contractual payments are more than 30 days past due. IFRS 9 also requires that (other than for purchased or
originated credit impaired financial instruments) if a significant increase in credit risk that had taken place
since initial recognition and has reversed by a subsequent reporting period (i.e., cumulatively credit risk is not
significantly higher than at initial recognition) then the expected credit losses on the financial instrument revert
to being measured based on an amount equal to the 12-month expected credit losses. [IFRS 9 paragraph 5.5.11]
Purchased or originated credit-impaired financial assets
Purchased or originated credit-impaired financial assets are treated differently because the asset is credit-
impaired at initial recognition. For these assets, an entity would recognise changes in lifetime expected losses
since initial recognition as a loss allowance with any changes recognised in profit or loss. Under the
requirements, any favourable changes for such assets are an impairment gain even if the resulting expected
cash flows of a financial asset exceed the estimated cash flows on initial recognition. [IFRS 9 paragraphs
5.5.13 – 5.5.14]
Credit-impaired financial asset
Under IFRS 9 a financial asset is credit-impaired when one or more events that have occurred and have a
significant impact on the expected future cash flows of the financial asset. It includes observable data that has
come to the attention of the holder of a financial asset about the following events:
[IFRS 9 Appendix A]
 significant financial difficulty of the issuer or borrower;
 a breach of contract, such as a default or past-due event;
 the lenders for economic or contractual reasons relating to the borrower’s financial difficulty granted
the borrower a concession that would not otherwise be considered;
 it becoming probable that the borrower will enter bankruptcy or other financial reorganisation;
 the disappearance of an active market for the financial asset because of financial difficulties; or
 the purchase or origination of a financial asset at a deep discount that reflects incurred credit losses.
Basis for estimating expected credit losses
Any measurement of expected credit losses under IFRS 9 shall reflect an unbiased and probability-weighted
amount that is determined by evaluating the range of possible outcomes as well as incorporating the time value
of money. Also, the entity should consider reasonable and supportable information about past events, current
conditions and reasonable and supportable forecasts of future economic conditions when measuring expected
credit losses. [IFRS 9 paragraph 5.5.17]
The Standard defines expected credit losses as the weighted average of credit losses with the respective risks
of a default occurring as the weightings. [IFRS 9 Appendix A] Whilst an entity does not need to consider
every possible scenario, it must consider the risk or probability that a credit loss occurs by considering the
possibility that a credit loss occurs and the possibility that no credit loss occurs, even if the probability of a
credit loss occurring is low. [IFRS 9 paragraph 5.5.18]
In particular, for lifetime expected losses, an entity is required to estimate the risk of a default occurring on
the financial instrument during its expected life. 12-month expected credit losses represent the lifetime cash
shortfalls that will result if a default occurs in the 12 months after the reporting date, weighted by the
probability of that default occurring.
An entity is required to incorporate reasonable and supportable information (i.e., that which is reasonably
available at the reporting date). Information is reasonably available if obtaining it does not involve undue cost
or effort (with information available for financial reporting purposes qualifying as such).
For applying the model to a loan commitment an entity will consider the risk of a default occurring under the
loan to be advanced, whilst application of the model for financial guarantee contracts an entity considers the
risk of a default occurring of the specified debtor. [IFRS 9 paragraphs B5.5.31 and B5.5.32]
An entity may use practical expedients when estimating expected credit losses if they are consistent with the
principles in the Standard (for example, expected credit losses on trade receivables may be calculated using a
provision matrix where a fixed provision rate applies depending on the number of days that a trade receivable
is outstanding). [IFRS 9 paragraph B5.5.35]
To reflect time value, expected losses should be discounted to the reporting date using the effective interest
rate of the asset (or an approximation thereof) that was determined at initial recognition. A “credit-adjusted
effective interest” rate should be used for expected credit losses of purchased or originated credit-impaired
financial assets. In contrast to the “effective interest rate” (calculated using expected cash flows that ignore
expected credit losses), the credit-adjusted effective interest rate reflects expected credit losses of the financial
asset. [IFRS 9 paragraphs B5.5.44-45]
Expected credit losses of undrawn loan commitments should be discounted by using the effective interest rate
(or an approximation thereof) that will be applied when recognising the financial asset resulting from the
commitment. If the effective interest rate of a loan commitment cannot be determined, the discount rate should
reflect the current market assessment of time value of money and the risks that are specific to the cash flows
but only if, and to the extent that, such risks are not taken into account by adjusting the discount rate. This
approach shall also be used to discount expected credit losses of financial guarantee contracts. [IFRS 9
paragraphs B5.5.47]
Presentation
Whilst interest revenue is always required to be presented as a separate line item, it is calculated differently
according to the status of the asset with regard to credit impairment. In the case of a financial asset that is not
a purchased or originated credit-impaired financial asset and for which there is no objective evidence of
impairment at the reporting date, interest revenue is calculated by applying the effective interest rate method
to the gross carrying amount. [IFRS 9 paragraph 5.4.1]
In the case of a financial asset that is not a purchased or originated credit-impaired financial asset but
subsequently has become credit-impaired, interest revenue is calculated by applying the effective interest rate
to the amortised cost balance, which comprises the gross carrying amount adjusted for any loss allowance.
[IFRS 9 paragraph 5.4.1]
In the case of purchased or originated credit-impaired financial assets, interest revenue is always recognised
by applying the credit-adjusted effective interest rate to the amortised cost carrying amount. [IFRS 9 paragraph
5.4.1] The credit-adjusted effective interest rate is the rate that discounts the cash flows expected on initial
recognition (explicitly taking account of expected credit losses as well as contractual terms of the instrument)
back to the amortised cost at initial recognition. [IFRS 9 Appendix A]
Consequential amendments of IFRS 9 to IAS 1 require that impairment losses, including reversals of
impairment losses and impairment gains (in the case of purchased or originated credit-impaired financial
assets), are presented in a separate line item in the statement of profit or loss and other comprehensive income.
Disclosures
IFRS 9 amends some of the requirements of IFRS 7 Financial Instruments: Disclosures including adding
disclosures about investments in equity instruments designated as at FVTOCI, disclosures on risk management
activities and hedge accounting and disclosures on credit risk management and impairment.
Interaction with IFRS 4
On 12 September 2016, the IASB issued amendments to IFRS 4 providing two options for entities that issue
insurance contracts within the scope of IFRS 4:
 an option that permits entities to reclassify, from profit or loss to other comprehensive income, some
of the income or expenses arising from designated financial assets; this is the so-called overlay
approach;

 an optional temporary exemption from applying IFRS 9 for entities whose predominant activity is
issuing contracts within the scope of IFRS 4; this is the so-called deferral approach.
An entity choosing to apply the overlay approach retrospectively to qualifying financial assets does so when
it first applies IFRS 9. An entity choosing to apply the deferral approach does so for annual periods beginning
on or after 1 January 2018. The application of both approaches is optional and an entity is permitted to stop
applying them before the new insurance contracts standard is applied.
1. The Polythene Pam Company purchases P2,000,000 of bonds. The asset has been designated as one at fair
value through profit and loss.
One year later, 10% of the bonds are sold for P400,000. Total cumulative gains previously recognized in
Polythene Pam's financial statements in respect of the asset are P100,000.
What is the amount of the gain on disposal to be recognized in profit or loss?

2. On January 1, 2020, SMB Company acquired the entire issue of Beerman’s P6,000,000 12% serial bonds.
The bonds were purchased to yield 10%. Bonds of P2,000,000 mature at annual intervals beginning December
31, 2020. Interest is payable annually on December 31. What is the carrying amount of the investment in
bonds on December 31, 2020?

3. On January 1, 2020, Alexander Corporation purchased P1,000,000 10% bonds for P927,880 (including
broker’s commission of P20,000). Alexander classified the bonds as FA@AC. The bonds were purchased to
yield 12%. Interest is payable annually every December 31. The bonds mature on December 31, 2024. On
December 31, 2020 the bonds were selling at 99. How much is the carrying amount the investment in bonds
on December 31, 2020?

4. On July 1, 2020, Morals Corp. acquired P4,000,000 face value of T Corporation bonds with a nominal rate
of interest of 4%. The bonds mature on July 1, 2025 and pay interest semi-annually each July 1 and January
1, with the first interest payment due on January 1, 2021. The bonds are classified as FA@AC. At the date of
issuance the bonds had a market rate of interest of 6%. The entity incurred transaction costs of 1% of the
purchase price. On December 31, 2020, the market value of the bonds was P3,700,000. The amount to be
recognized in 2020 profit or loss related to the bond investment is

5. On April 1, 2020, Saxe, Inc. purchased P2,000,000 face value, 9%, Treasury Notes for P1,985,000,
including accrued interest of P45,000. The notes mature on July 1, 2021, and pay interest semiannually on
January 1 and July 1. Saxe uses the straight-line method of amortization. If the notes are classified as FA@AC,
the carrying amount of this investment in the company’s October 31, 2020 statement of financial position
should be

6. On January 1, 2020, Choson Corporation purchased P4,000,000 10% bonds for P3,711,520. These bonds
are held in a business model whose objective is achieved by both collecting contractual cash flows and selling
financial assets. The bonds were purchased to yield 12%. Interest is payable annually every December 31. The
bonds mature on December 31, 2024. On December 31, 2020 the bonds were selling at 99. On December 31,
2020, Choson sold P2,000,000 face value bonds at 101, which is the fair value of the bonds on that date, plus
accrued interest. The unrealized gain to be recognized as a separate component of equity on December 31,
2020 is

7. The gain on sale of the bonds on December 31, 2021 is

8. On June 30, 2020, Aileen Corp. purchased a two-year bond, which it classified as FA@FVTOCI. The bond
had a stated principal amount of P10,000,000, which Aileen Corp. will receive on June 30, 2022. The stated
coupon interest rate was 10% per year, which is paid
semiannually on December 31 and June 30. The bond was purchased at a quoted annual yield of 8% on a
bond-equivalent yield basis. On December 31, 2020, the bonds are quoted at 101.1. How much should be
recognized as component of equity as of December 31, 2020 related to this bond investment? (Round off
present value factors to four decimal places)
Determine whether these statements relate to you. Answer Yes No
honestly to check your progress.
I am able to define debt securities and related terms.
I can identify debt securities from other types of investments.
I understand the measuring standard for debt securities.
I can determine when to derecognize debt securities from the books.
I can determine the presentation and disclosure requirements for
debt securities.
Total (Raw Score)
Definitions
IAS 39 incorporates the definitions of the following items from IAS 32 Financial Instruments: Presentation:
[IAS 39.8]
 financial instrument
 financial asset
 financial liability
 equity instrument.
Note: Where an entity applies IFRS 9 Financial Instruments prior to its mandatory application date (1
January 2015), definitions of the following terms are also incorporated from IFRS 9: derecognition,
derivative, fair value, financial guarantee contract. The definition of those terms outlined below (as
relevant) are those from IAS 39.

These questions help you recognize your existing background Yes No


knowledge on the topic. Answer honestly.
Does the standard definition given above match your personal
definition for investments?
Can you differentiate the different types of investments?
Have you read the standard for measuring investments?
Can you determine when to derecognize investments from the
books?
Have you read the standard for presentation and disclosure of
investments?
Total (Raw Score)

IFRS 7 Financial Instruments: Disclosures (summary) by Silvia of CPDbox.


View at: https://www.youtube.com/watch?v=uEwoWTQmd4k

Overview of IFRS 7
IFRS 7:
 adds certain new disclosures about financial instruments to those previously required
by IAS 32 Financial Instruments: Disclosure and Presentation (as it was then cited)
 replaces the disclosures previously required by IAS 30 Disclosures in the Financial Statements of
Banks and Similar Financial Institutions
 puts all of those financial instruments disclosures together in a new standard on Financial
Instruments: Disclosures. The remaining parts of IAS 32 deal only with financial instruments
presentation matters.
Disclosure requirements of IFRS 7
IFRS requires certain disclosures to be presented by category of instrument based on the IAS 39
measurement categories. Certain other disclosures are required by class of financial instrument. For those
disclosures an entity must group its financial instruments into classes of similar instruments as appropriate to
the nature of the information presented. [IFRS 7.6]
The two main categories of disclosures required by IFRS 7 are:
1. information about the significance of financial instruments.
2. information about the nature and extent of risks arising from financial instruments
Information about the significance of financial instruments
Statement of financial position
 Disclose the significance of financial instruments for an entity's financial position and performance.
[IFRS 7.7] This includes disclosures for each of the following categories: [IFRS 7.8]
o financial assets measured at fair value through profit and loss, showing separately those held
for trading and those designated at initial recognition
o held-to-maturity investments
o loans and receivables
o available-for-sale assets
o financial liabilities at fair value through profit and loss, showing separately those held for
trading and those designated at initial recognition
o financial liabilities measured at amortised cost
 Other balance sheet-related disclosures:
o special disclosures about financial assets and financial liabilities designated to be measured at
fair value through profit and loss, including disclosures about credit risk and market risk,
changes in fair values attributable to these risks and the methods of measurement.[IFRS 7.9-
11]
o reclassifications of financial instruments from one category to another (e.g. from fair value to
amortised cost or vice versa) [IFRS 7.12-12A]
o information about financial assets pledged as collateral and about financial or non-financial
assets held as collateral [IFRS 7.14-15]
o reconciliation of the allowance account for credit losses (bad debts) by class of financial
assets[IFRS 7.16]
o information about compound financial instruments with multiple embedded derivatives
[IFRS 7.17]
o breaches of terms of loan agreements [IFRS 7.18-19]
Statement of comprehensive income
 Items of income, expense, gains, and losses, with separate disclosure of gains and losses from: [IFRS
7.20(a)]
o financial assets measured at fair value through profit and loss, showing separately those held
for trading and those designated at initial recognition.
o held-to-maturity investments.
o loans and receivables.
o available-for-sale assets.
o financial liabilities measured at fair value through profit and loss, showing separately those
held for trading and those designated at initial recognition.
o financial liabilities measured at amortised cost.
 Other income statement-related disclosures:
o total interest income and total interest expense for those financial instruments that are not
measured at fair value through profit and loss [IFRS 7.20(b)]
o fee income and expense [IFRS 7.20(c)]
o amount of impairment losses by class of financial assets [IFRS 7.20(e)]
o interest income on impaired financial assets [IFRS 7.20(d)]
Other disclosures
 Accounting policies for financial instruments [IFRS 7.21]
 Information about hedge accounting, including: [IFRS 7.22]
o description of each hedge, hedging instrument, and fair values of those instruments, and
nature of risks being hedged
o for cash flow hedges, the periods in which the cash flows are expected to occur, when they
are expected to enter into the determination of profit or loss, and a description of any forecast
transaction for which hedge accounting had previously been used but which is no longer
expected to occur
o if a gain or loss on a hedging instrument in a cash flow hedge has been recognised in other
comprehensive income, an entity should disclose the following: [IAS 7.23]
o the amount that was so recognised in other comprehensive income during the period
o the amount that was removed from equity and included in profit or loss for the period
o the amount that was removed from equity during the period and included in the initial
measurement of the acquisition cost or other carrying amount of a non-financial asset or non-
financial liability in a hedged highly probable forecast transaction
Note: Where IFRS 9 Financial Instruments (2013) is applied, revised disclosure requirements
apply. The required hedge accounting disclosures apply where the entity elects to adopt
hedge accounting and require information to be provided in three broad categories: (1) the
entity’s risk management strategy and how it is applied to manage risk (2) how the entity’s
hedging activities may affect the amount, timing and uncertainty of its future cash flows, and
(3) the effect that hedge accounting has had on the entity’s statement of financial position,
statement of comprehensive income and statement of changes in equity. The disclosures are
required to be presented in a single note or separate section in its financial statements,
although some information can be incorporated by reference.
 For fair value hedges, information about the fair value changes of the hedging instrument and the
hedged item [IFRS 7.24(a)]
 Hedge ineffectiveness recognised in profit and loss (separately for cash flow hedges and hedges of a
net investment in a foreign operation) [IFRS 7.24(b-c)]
 Uncertainty arising from the interest rate benchmark reform [IFRS 7.24H]
 Information about the fair values of each class of financial asset and financial liability, along with:
[IFRS 7.25-30]
o comparable carrying amounts
o description of how fair value was determined
o the level of inputs used in determining fair value
o reconciliations of movements between levels of fair value measurement hierarchy additional
disclosures for financial instruments whose fair value is determined using level 3 inputs
including impacts on profit and loss, other comprehensive income and sensitivity analysis
o information if fair value cannot be reliably measured
The fair value hierarchy introduces 3 levels of inputs based on the lowest level of input significant to the
overall fair value (IFRS 7.27A-27B):
 Level 1 – quoted prices for similar instruments
 Level 2 – directly observable market inputs other than Level 1 inputs
 Level 3 – inputs not based on observable market data
Note that disclosure of fair values is not required when the carrying amount is a reasonable approximation of
fair value, such as short-term trade receivables and payables, or for instruments whose fair value cannot be
measured reliably. [IFRS 7.29(a)]
Nature and extent of exposure to risks arising from financial instruments
Qualitative disclosures [IFRS 7.33]
 The qualitative disclosures describe:
o risk exposures for each type of financial instrument
o management's objectives, policies, and processes for managing those risks
o changes from the prior period
Quantitative disclosures
 The quantitative disclosures provide information about the extent to which the entity is exposed to
risk, based on information provided internally to the entity's key management personnel. These
disclosures include: [IFRS 7.34]
o summary quantitative data about exposure to each risk at the reporting date
o disclosures about credit risk, liquidity risk, and market risk and how these risks are managed
as further described below
o concentrations of risk
Credit risk
 Credit risk is the risk that one party to a financial instrument will cause a loss for the other party by
failing to pay for its obligation. [IFRS 7. Appendix A]
 Disclosures about credit risk include: [IFRS 7.36-38]
o maximum amount of exposure (before deducting the value of collateral), description of
collateral, information about credit quality of financial assets that are neither past due nor
impaired, and information about credit quality of financial assets whose terms have been
renegotiated [IFRS 7.36]
o for financial assets that are past due or impaired, analytical disclosures are required [IFRS
7.37]
o information about collateral or other credit enhancements obtained or called [IFRS 7.38]
Liquidity risk
 Liquidity risk is the risk that an entity will have difficulties in paying its financial liabilities. [IFRS 7.
Appendix A]
 Disclosures about liquidity risk include: [IFRS 7.39]
o a maturity analysis of financial liabilities
o description of approach to risk management
Market risk [IFRS 7.40-42]
 Market risk is the risk that the fair value or cash flows of a financial instrument will fluctuate due to
changes in market prices. Market risk reflects interest rate risk, currency risk and other price risks.
[IFRS 7. Appendix A]
 Disclosures about market risk include:
o a sensitivity analysis of each type of market risk to which the entity is exposed
o additional information if the sensitivity analysis is not representative of the entity's risk
exposure (for example because exposures during the year were different to exposures at year-
end).
o IFRS 7 provides that if an entity prepares a sensitivity analysis such as value-at-risk for
management purposes that reflects interdependencies of more than one component of market
risk (for instance, interest risk and foreign currency risk combined), it may disclose that
analysis instead of a separate sensitivity analysis for each type of market risk
Transfers of financial assets [IFRS 7.42A-H]
An entity shall disclose information that enables users of its financial statements:
1. to understand the relationship between transferred financial assets that are not derecognised in their
entirety and the associated liabilities; and
2. to evaluate the nature of, and risks associated with, the entity's continuing involvement in
derecognised financial assets. [IFRS 7 42B]
Transferred financial assets that are not derecognised in their entirety
 Required disclosures include description of the nature of the transferred assets, nature of risk and
rewards as well as description of the nature and quantitative disclosure depicting relationship
between transferred financial assets and the associated liabilities. [IFRS 7.42D]
Transferred financial assets that are derecognised in their entirety
 Required disclosures include the carrying amount of the assets and liabilities recognised, fair value
of the assets and liabilities that represent continuing involvement, maximum exposure to loss from
the continuing involvement as well as maturity analysis of the undiscounted cash flows to repurchase
the derecognised financial assets. [IFRS 7.42E]
 Additional disclosures are required for any gain or loss recognised at the date of transfer of the
assets, income or expenses recognise from the entity's continuing involvement in the derecognised
financial assets as well as details of uneven distribution of proceed from transfer activity throughout
the reporting period. [IFRS 7.42G]
1. On Feb. 2, 2019, I AM DETERMINED CO. purchased 10,000 shares of CPA CO. at P56 plus broker’s
commission of P4 per share. The investment is FVTOCI. During 2018 and 2020, the following events occurred
regarding the investment:
12/15/19 CPA CO. declares and pays a P2.20 per share dividend
12/31/19 The market price of CPA CO. stock is P52 per share at year-end
12/01/20 CPA CO. declares and pays a dividend of P2 per share
12/31/20 The market price of CPA CO. stock is P55 per share at year-end
The net unrealized loss at December 31, 2020 in accumulated OCI in shareholders' equity is

Use the following information for the next two questions.


On December 28, 2020, Anne Company commits itself to purchase a financial asset to be classified as FVTPL
for P800,000, its fair value on commitment (trade) date. This security has a fair value of P801,000 and
P802,500 on December 31, 2020 (Anne's financial year-end), and
January 5, 2021 (settlement date), respectively.

2. If Anne applies the trade date accounting method to account for regular way purchases of its securities, how
much gain should be recognized on January 5, 2021?

3. If Anne applies the settlement date accounting method to account for regular way purchases of its securities,
how much gain should be recognized on January 5, 2021?

4. On December 28, 2020, Bakeks Company commits itself to purchase equity securities to be classified as
held for trading for P1,000,000, its fair value on commitment (trade) date. These securities have a fair value
of P1,002,000 and P1,005,000 on December 31, 2020 (Bakeks' financial year-end), and January 5, 2021
(settlement date), respectively. If Bakeks applies the settlement date accounting method to account for regular-
way purchases, how much should
be recognized in its 2020 profit or loss related to these securities?

5. On December 28, 2020 (trade date), Charming Corp. enters into a contract to sell an equity security
classified as FA@FVTOCI for its current fair value of P505,000. The asset was acquired a year ago and its
cost was P500,000. On December 31, 2020 (financial year-end), the fair value of the asset is P506,000. On
January 5, 2021 (settlement date), the asset's fair value is P507,500. If Charming uses the trade date method
to account for regular-way sales of its securities, how much should be reported as reclassification adjustment
in its 2020 financial
statements?

Determine whether these statements relate to you. Answer Yes No


honestly to check your progress.
I am able to define investments and related terms.
I can identify investments from other types of assets.
I can differentiate the types of investments.
I understand the measuring standard for investments.
I can determine when to derecognize investments from the books.
I can determine the presentation and disclosure requirements for
investments.
Total (Raw Score)
Associate An entity over which the investor has significant influence

Significant The power to participate in the financial and operating policy


influence decisions of the investee but is not control or joint control of
those policies

Joint An arrangement of which two or more parties have joint control


arrangement

Joint control The contractually agreed sharing of control of an arrangement,


which exists only when decisions about the relevant activities
require the unanimous consent of the parties sharing control

Joint venture A joint arrangement whereby the parties that have joint control
of the arrangement have rights to the net assets of the
arrangement

Joint A party to a joint venture that has joint control of that joint
venturer venture

Equity A method of accounting whereby the investment is initially


method recognised at cost and adjusted thereafter for the post-acquisition
change in the investor's share of the investee's net assets. The
investor's profit or loss includes its share of the investee's profit
or loss and the investor's other comprehensive income includes
its share of the investee's other comprehensive income

These questions help you recognize your existing background Yes No


knowledge on the topic. Answer honestly.
Does the standard definition given above match your personal
definition for investment in associates?
Can you differentiate investments in associates from the other types
of investments?
Have you read the standard for measuring investments in
associates?
Can you determine when to derecognize investments in associates
from the books?
Have you read the standard for presentation and disclosure of
investments in associates?
Total (Raw Score)

IAS 28 Investments in Associates and Joint Ventures by Silvia of CPDbox.


View at: https://www.youtube.com/watch?v=Krc3jlO-kZI

IAS 28 — Investments in Associates and Joint Ventures (2011)


Scope of IAS 28
IAS 28 applies to all entities that are investors with joint control of, or significant influence over, an investee
(associate or joint venture). [IAS 28(2011).2]
Significant influence
Where an entity holds 20% or more of the voting power (directly or through subsidiaries) on an investee, it
will be presumed the investor has significant influence unless it can be clearly demonstrated that this is not
the case. If the holding is less than 20%, the entity will be presumed not to have significant influence unless
such influence can be clearly demonstrated. A substantial or majority ownership by another investor does not
necessarily preclude an entity from having significant influence. [IAS 28(2011).5]
The existence of significant influence by an entity is usually evidenced in one or more of the following ways:
[IAS 28(2011).6]
 representation on the board of directors or equivalent governing body of the investee;

 participation in the policy-making process, including participation in decisions about dividends or


other distributions;

 material transactions between the entity and the investee;

 interchange of managerial personnel; or

 provision of essential technical information


The existence and effect of potential voting rights that are currently exercisable or convertible, including
potential voting rights held by other entities, are considered when assessing whether an entity has significant
influence. In assessing whether potential voting rights contribute to significant influence, the entity examines
all facts and circumstances that affect potential rights [IAS 28(2011).7, IAS 28(2011).8]
An entity loses significant influence over an investee when it loses the power to participate in the financial
and operating policy decisions of that investee. The loss of significant influence can occur with or without a
change in absolute or relative ownership levels. [IAS 28(2011).9]
The equity method of accounting
Basic principle. Under the equity method, on initial recognition the investment in an associate or a joint
venture is recognised at cost, and the carrying amount is increased or decreased to recognise the investor's
share of the profit or loss of the investee after the date of acquisition. [IAS 28(2011).10]
Distributions and other adjustments to carrying amount. The investor's share of the investee's profit or
loss is recognised in the investor's profit or loss. Distributions received from an investee reduce the carrying
amount of the investment. Adjustments to the carrying amount may also be necessary for changes in the
investor's proportionate interest in the investee arising from changes in the investee's other comprehensive
income (e.g. to account for changes arising from revaluations of property, plant and equipment and foreign
currency translations.) [IAS 28(2011).10]
Potential voting rights. An entity's interest in an associate or a joint venture is determined solely on the basis
of existing ownership interests and, generally, does not reflect the possible exercise or conversion of potential
voting rights and other derivative instruments. [IAS 28(2011).12]
Interaction with IFRS 9. IFRS 9 Financial Instruments does not apply to interests in associates and joint
ventures that are accounted for using the equity method. An entity applies IFRS 9, including its impairment
requirements, to long-term interests in an associate or joint venture that form part of the net investment in the
associate or joint venture but to which the equity method is not applied. Instruments containing potential
voting rights in an associate or a joint venture are accounted for in accordance with IFRS 9, unless they
currently give access to the returns associated with an ownership interest in an associate or a joint venture.
[IAS 28(2011).14-14A]
Classification as non-current asset. An investment in an associate or a joint venture is generally classified
as non-current asset, unless it is classified as held for sale in accordance with IFRS 5 Non-current Assets Held
for Sale and Discontinued Operations. [IAS 28(2011).15]
Application of the equity method of accounting
Basic principle. In its consolidated financial statements, an investor uses the equity method of accounting for
investments in associates and joint ventures. [IAS 28(2011).16] Many of the procedures that are appropriate
for the application of the equity method are similar to the consolidation procedures described in IFRS 10.
Furthermore, the concepts underlying the procedures used in accounting for the acquisition of a subsidiary are
also adopted in accounting for the acquisition of an investment in an associate or a joint venture. [IAS
28.(2011).26]
Exemptions from applying the equity method. An entity is exempt from applying the equity method if the
investment meets one of the following conditions:
 The entity is a parent that is exempt from preparing consolidated financial statements under IFRS
10 Consolidated Financial Statementsor or if all of the following four conditions are met (in which
case the entity need not apply the equity method): [IAS 28(2011).17]
o the entity is a wholly-owned subsidiary, or is a partially-owned subsidiary of another entity and
its other owners, including those not otherwise entitled to vote, have been informed about, and
do not object to, the investor not applying the equity method
o the investor or joint venturer's debt or equity instruments are not traded in a public market
o the entity did not file, nor is it in the process of filing, its financial statements with a securities
commission or other regulatory organisation for the purpose of issuing any class of instruments
in a public market, and
o the ultimate or any intermediate parent of the parent produces financial statements available
for public use that comply with IFRSs, in which subsidiaries are consolidated or are measured
at fair value through profit or loss in accordance with IFRS 10.
 When an investment in an associate or a joint venture is held by, or is held indirectly through, an entity
that is a venture capital organisation, or a mutual fund, unit trust and similar entities including
investment-linked insurance funds, the entity may elect to measure investments in those associates and
joint ventures at fair value through profit or loss in accordance with IFRS 9. The election is made
separately for each associate or joint venture on initial recognition. [IAS 28(2011).18] When an entity
has an investment in an associate, a portion of which is held indirectly through a venture capital
organisation, or a mutual fund, unit trust and similar entities including investment-linked insurance
funds, the entity may elect to measure that portion of the investment in the associate at fair value
through profit or loss in accordance with IFRS 9 regardless of whether the venture capital organisation,
or the mutual fund, unit trust and similar entities including investment-linked insurance funds, has
significant influence over that portion of the investment. If the entity makes that election, the entity
shall apply the equity method to any remaining portion of its investment in an associate that is not held
through a venture capital organisation, or a mutual fund, unit trust and similar entities including
investment-linked insurance funds. [IAS 28(2011).19]
Classification as held for sale. When the investment, or portion of an investment, meets the criteria to be
classified as held for sale, the portion so classified is accounted for in accordance with IFRS 5. Any remaining
portion is accounted for using the equity method until the time of disposal, at which time the retained
investment is accounted under IFRS 9, unless the retained interest continues to be an associate or joint venture.
[IAS 28(2011).20]
Discontinuing the equity method. Use of the equity method should cease from the date that significant
influence or joint control ceases: [IAS 28(2011).22]
 If the investment becomes a subsidiary, the entity accounts for its investment in accordance with IFRS
3 Business Combinations and IFRS 10
 If the retained interest is a financial asset, it is measured at fair value and subsequently accounted for
under IFRS 9

 Any amounts recognised in other comprehensive income in relation to the investment in the associate
or joint venture are accounted for on the same basis as if the investee had directly disposed of the
related assets or liabilities (which may require reclassification to profit or loss)
 If an investment in an associate becomes an investment in a joint venture (or vice versa), the entity
continues to apply the equity method and does not remeasure the retained interest. [IAS 28(2011).24]
Changes in ownership interests. If an entity's interest in an associate or joint venture is reduced, but the
equity method is continued to be applied, the entity reclassifies to profit or loss the proportion of the gain or
loss previously recognised in other comprehensive income relative to that reduction in ownership interest.
[IAS 28(2011).25]
Equity method procedures.
 Transactions with associates or joint ventures. Profits and losses resulting from upstream (associate
to investor, or joint venture to joint venturer) and downstream (investor to associate, or joint venturer
to joint venture) transactions are eliminated to the extent of the investor's interest in the associate or
joint venture. However, unrealised losses are not eliminated to the extent that the transaction provides
evidence of a reduction in the net realisable value or in the recoverable amount of the assets transferred.
Contributions of non-monetary assets to an associate or joint venture in exchange for an equity interest
in the associate or joint venture are also accounted for in accordance with these requirements. [IAS
28(2011).28-30]
 Initial accounting. An investment is accounted for using the equity method from the date on which it
becomes an associate or a joint venture. On acquisition, any difference between the cost of the
investment and the entity’s share of the net fair value of the investee's identifiable assets and liabilities
in case of goodwill is included in the carrying amount of the investment (amortisation not permitted)
and any excess of the entity's share of the net fair value of the investee's identifiable assets and
liabilities over the cost of the investment is included as income in the determination of the entity's
share of the associate or joint venture’s profit or loss in the period in which the investment is acquired.
Adjustments to the entity's share of the associate's or joint venture's profit or loss after acquisition are
made, for example, for depreciation of the depreciable assets based on their fair values at the
acquisition date or for impairment losses such as for goodwill or property, plant and equipment. [IAS
28(2011).32]
 Date of financial statements. In applying the equity method, the investor or joint venturer should use
the financial statements of the associate or joint venture as of the same date as the financial statements
of the investor or joint venturer unless it is impracticable to do so. If it is impracticable, the most recent
available financial statements of the associate or joint venture should be used, with adjustments made
for the effects of any significant transactions or events occurring between the accounting period ends.
However, the difference between the reporting date of the associate and that of the investor cannot be
longer than three months. [IAS 28(2011).33, IAS 28(2011).34]
 Accounting policies. If the associate or joint venture uses accounting policies that differ from those of
the investor, the associate or joint venture's financial statements are adjusted to reflect the investor's
accounting policies for the purpose of applying the equity method. [IAS 28(2011).35]
 Application of the equity method by a non-investment entity investor to an investment entity
investee. When applying the equity method to an associate or a joint venture, a non-investment entity
investor in an investment entity may retain the fair value measurement applied by the associate or joint
venture to its interests in subsidiaries. The election is made separately for each associate or joint
venture.[IAS 28(2011).36A]
 Losses in excess of investment. If an investor's or joint venturer's share of losses of an associate or joint
venture equals or exceeds its interest in the associate or joint venture, the investor or joint venturer
discontinues recognising its share of further losses. The interest in an associate or joint venture is the
carrying amount of the investment in the associate or joint venture under the equity method together
with any long-term interests that, in substance, form part of the investor or joint venturer's net
investment in the associate or joint venture. After the investor or joint venturer's interest is reduced to
zero, a liability is recognised only to the extent that the investor or joint venturer has incurred legal or
constructive obligations or made payments on behalf of the associate. If the associate or joint venture
subsequently reports profits, the investor or joint venturer resumes recognising its share of those profits
only after its share of the profits equals the share of losses not recognised. [IAS 28(2011).38, IAS
28(2011).39]
Impairment. After application of the equity method an entity applies IAS 39 Financial Instruments:
Recognition and Measurement to determine whether it is necessary to recognise any additional impairment
loss with respect to its net investment in the associate or joint venture. If impairment is indicated, the amount
is calculated by reference to IAS 36 Impairment of Assets. The entire carrying amount of the investment is
tested for impairment as a single asset, that is, goodwill is not tested separately. The recoverable amount of an
investment in an associate is assessed for each individual associate or joint venture, unless the associate or
joint venture does not generate cash flows independently. [IAS 28(2011).40, IAS 28(2011).42, IAS
28(2011).43]
Separate financial statements
An investment in an associate or a joint venture shall be accounted for in the entity's separate financial
statements in accordance with IAS 27 Separate Financial Statements (as amended in 2011).
Disclosure
There are no disclosures specified in IAS 28. Instead, IFRS 12 Disclosure of Interests in Other
Entities outlines the disclosures required for entities with joint control of, or significant influence over, an
investee.
Applicability and early adoption
IAS 28 (2011) is applicable to annual reporting periods beginning on or after 1 January 2013. [IAS
28(2011).45]
An entity may apply IAS 28 (2011) to an earlier accounting period, but if doing so it must disclose the fact
that is has early adopted the standard and also apply: [IAS 28(2011).45]
 IFRS 10 Consolidated Financial Statements
 IFRS 11 Joint Arrangements
 IFRS 12 Disclosure of Interests in Other Entities
 IAS 27 Separate Financial Statements (2011).

1. On July 1, 2020, Diamond, Inc, paid P1,000,000 for 100,000 ordinary shares (40%) of Ashley Corporation.
At that date the net assets of Ashley totaled P2,500,000 and the fair values of all of Ashley's identifiable assets
and liabilities were equal to their book values. Ashley reported profit of P500,000 for the year ended December
31, 2020, of which P300,000 was for the six months ended December 31, 2020. Ashley paid cash dividends
of P250,000 on September 30, 2020. Diamond does not elect the fair value option for reporting its investment
in Ashley. In its income statement for the year ended December 31, 2020, what amount of income should
Diamond report from its investments in Ashley?

2. On December 1, 2020, Citrus purchased 200,000 shares representing 45 percent of the outstanding stock of
Berry for cash of P2,500,000. As a result of this purchase, Citrus has the ability to exercise significant
influence over the operating and financial policies of Berry. 45 percent of the profit of Berry amounted to
P20,000 for the month of December and P350,000 for the year ended December 31, 2020. On January 15,
2021, cash dividends of P0.30 per share
were paid to shareholders of record on December 31, 2020. Citrus' longterm investment in Berry should be
shown in Citrus' December 31, 2020, statement of financial position at:

3. On January 1, 2020, Solana Co. purchased 25% of Orr Corp.'s ordinary shares; no goodwill resulted from
the purchase. Solana appropriately carries this investment at equity and the balance in Solana’s investment
account was P480,000 at December 31, 2020. Orr reported profit of P300,000 for the year ended December
31, 2020, and paid dividends totaling P120,000 during 2020. How much did Solana pay for its 25% interest
in Orr?

4. Investor company acquired a 40% interest in an associate for P3,000,000. In the financial period
immediately following the date on which it became an associate, the investee took the following action:
• revalued assets up to fair value by P500,000
• generated profits of P1,600,000
• declared a dividend of P300,000
The balance in the investor’s account of ‘Shares in associate’, after equity accounting has been applied, is:

5. Baggao Company purchased 15% of Badoc Company’s 500,000 outstanding ordinary shares on January 2,
2020, for P15,000,000. On December 31, 2020, Baggao purchased additional 125,000 shares of Badoc for
P35,000,000. Badoc reported earnings of P20,000,000 for 2020. What amount should Baggao report in its
December 31, 2020 statement of financial position as investment in Badoc Company?
Determine whether these statements relate to you. Answer Yes No
honestly to check your progress.
I am able to define investment in associates and related terms.
I can identify investment in associates from other types of assets.
I understand the measuring standard for investment in associates.
I can determine when to derecognize investment in associates from
the books.
I can determine the presentation and disclosure requirements for
investment in associates.
Total (Raw Score)
IAS 39 incorporates the definitions of the following items from IAS 32 Financial Instruments: Presentation:
[IAS 39.8]
 financial instrument
 financial asset
 financial liability
 equity instrument.
Note: Where an entity applies IFRS 9 Financial Instruments prior to its mandatory application date (1
January 2015), definitions of the following terms are also incorporated from IFRS 9: derecognition,
derivative, fair value, financial guarantee contract. The definition of those terms outlined below (as
relevant) are those from IAS 39.

These questions help you recognize your existing background Yes No


knowledge on the topic. Answer honestly.
Does the standard definition given above match your personal
definition for derivatives?
Can you differentiate derivatives from the other types of
investments?
Have you read the standard for measuring derivatives?
Can you determine when to derecognize derivatives from the
books?
Have you read the standard for presentation and disclosure of
derivatives?
Total (Raw Score)

Hedge Accounting IAS 39 vs. IFRS 9 by Silvia of CPDbox.


View at: https://www.youtube.com/watch?v=RGgI1mYVSEs

IFRS 9 — Financial Instruments


Derivatives
All derivatives in scope of IFRS 9, including those linked to unquoted equity investments, are measured at
fair value. Value changes are recognised in profit or loss unless the entity has elected to apply hedge
accounting by designating the derivative as a hedging instrument in an eligible hedging relationship.
Embedded derivatives
An embedded derivative is a component of a hybrid contract that also includes a non-derivative host, with the
effect that some of the cash flows of the combined instrument vary in a way similar to a stand-alone derivative.
A derivative that is attached to a financial instrument but is contractually transferable independently of that
instrument, or has a different counterparty, is not an embedded derivative, but a separate financial instrument.
[IFRS 9, paragraph 4.3.1]
The embedded derivative concept that existed in IAS 39 has been included in IFRS 9 to apply only to hosts
that are not financial assets within the scope of the Standard. Consequently, embedded derivatives that under
IAS 39 would have been separately accounted for at FVTPL because they were not closely related to the host
financial asset will no longer be separated. Instead, the contractual cash flows of the financial asset are
assessed in their entirety, and the asset as a whole is measured at FVTPL if the contractual cash flow
characteristics test is not passed (see above).
The embedded derivative guidance that existed in IAS 39 is included in IFRS 9 to help preparers identify
when an embedded derivative is closely related to a financial liability host contract or a host contract not
within the scope of the Standard (e.g. leasing contracts, insurance contracts, contracts for the purchase or sale
of a non-financial items).
Reclassification
For financial assets, reclassification is required between FVTPL, FVTOCI and amortised cost, if and only if
the entity's business model objective for its financial assets changes so its previous model assessment would
no longer apply. [IFRS 9, paragraph 4.4.1]
If reclassification is appropriate, it must be done prospectively from the reclassification date which is defined
as the first day of the first reporting period following the change in business model. An entity does not restate
any previously recognised gains, losses, or interest.
IFRS 9 does not allow reclassification:
 for equity investments measured at FVTOCI, or
 where the fair value option has been exercised in any circumstance for a financial assets or financial
liability.
Hedge accounting
The hedge accounting requirements in IFRS 9 are optional. If certain eligibility and qualification criteria are
met, hedge accounting allows an entity to reflect risk management activities in the financial statements by
matching gains or losses on financial hedging instruments with losses or gains on the risk exposures they
hedge.
The hedge accounting model in IFRS 9 is not designed to accommodate hedging of open, dynamic portfolios.
As a result, for a fair value hedge of interest rate risk of a portfolio of financial assets or liabilities an entity
can apply the hedge accounting requirements in IAS 39 instead of those in IFRS 9. [IFRS 9 paragraph 6.1.3]
In addition when an entity first applies IFRS 9, it may choose as its accounting policy choice to continue to
apply the hedge accounting requirements of IAS 39 instead of the requirements of Chapter 6 of IFRS 9 [IFRS
9 paragraph 7.2.21]
Qualifying criteria for hedge accounting
A hedging relationship qualifies for hedge accounting only if all of the following criteria are met:
1. the hedging relationship consists only of eligible hedging instruments and eligible hedged items.
2. at the inception of the hedging relationship there is formal designation and documentation of the
hedging relationship and the entity’s risk management objective and strategy for undertaking the
hedge.
3. the hedging relationship meets all of the hedge effectiveness requirements (see below) [IFRS 9
paragraph 6.4.1]
Hedging instruments
Only contracts with a party external to the reporting entity may be designated as hedging instruments. [IFRS
9 paragraph 6.2.3]
A hedging instrument may be a derivative (except for some written options) or non-derivative financial
instrument measured at FVTPL unless it is a financial liability designated as at FVTPL for which changes due
to credit risk are presented in OCI. For a hedge of foreign currency risk, the foreign currency risk component
of a non-derivative financial instrument, except equity investments designated as FVTOCI, may be designated
as the hedging instrument. [IFRS 9 paragraphs 6.2.1-6.2.2]
IFRS 9 allows a proportion (e.g. 60%) but not a time portion (eg the first 6 years of cash flows of a 10 year
instrument) of a hedging instrument to be designated as the hedging instrument. IFRS 9 also allows only the
intrinsic value of an option, or the spot element of a forward to be designated as the hedging instrument. An
entity may also exclude the foreign currency basis spread from a designated hedging instrument. [IFRS 9
paragraph 6.2.4]
IFRS 9 allows combinations of derivatives and non-derivatives to be designated as the hedging instrument.
[IFRS 9 paragraph 6.2.5]
Combinations of purchased and written options do not qualify if they amount to a net written option at the
date of designation. [IFRS 9 paragraph 6.2.6]
Hedged items
A hedged item can be a recognised asset or liability, an unrecognised firm commitment, a highly probable
forecast transaction or a net investment in a foreign operation and must be reliably measurable. [IFRS 9
paragraphs 6.3.1-6.3.3]
An aggregated exposure that is a combination of an eligible hedged item as described above and a derivative
may be designated as a hedged item. [IFRS 9 paragraph 6.3.4]
The hedged item must generally be with a party external to the reporting entity, however, as an exception the
foreign currency risk of an intragroup monetary item may qualify as a hedged item in the consolidated
financial statements if it results in an exposure to foreign exchange rate gains or losses that are not fully
eliminated on consolidation. In addition, the foreign currency risk of a highly probable forecast intragroup
transaction may qualify as a hedged item in consolidated financial statements provided that the transaction is
denominated in a currency other than the functional currency of the entity entering into that transaction and
the foreign currency risk will affect consolidated profit or loss. [IFRS 9 paragraphs 6.3.5 -6.3.6]
An entity may designate an item in its entirety or a component of an item as the hedged item. The component
may be a risk component that is separately identifiable and reliably measurable; one or more selected
contractual cash flows; or components of a nominal amount. [IFRS 9 paragraph 6.3.7]
A group of items (including net positions is an eligible hedged item only if:
1. it consists of items individually, eligible hedged items;
2. the items in the group are managed together on a group basis for risk management purposes; and
3. in the case of a cash flow hedge of a group of items whose variabilities in cash flows are not expected
to be approximately proportional to the overall variability in cash flows of the group:
1. it is a hedge of foreign currency risk; and
2. the designation of that net position specifies the reporting period in which the forecast
transactions are expected to affect profit or loss, as well as their nature and volume [IFRS 9
paragraph 6.6.1]
For a hedge of a net position whose hedged risk affects different line items in the statement of profit or loss
and other comprehensive income, any hedging gains or losses in that statement are presented in a separate line
from those affected by the hedged items. [IFRS 9 paragraph 6.6.4]
Accounting for qualifying hedging relationships
There are three types of hedging relationships:
Fair value hedge: a hedge of the exposure to changes in fair value of a recognised asset or liability or an
unrecognised firm commitment, or a component of any such item, that is attributable to a particular risk and
could affect profit or loss (or OCI in the case of an equity instrument designated as at FVTOCI). [IFRS 9
paragraphs 6.5.2(a) and 6.5.3]
For a fair value hedge, the gain or loss on the hedging instrument is recognised in profit or loss (or OCI, if
hedging an equity instrument at FVTOCI and the hedging gain or loss on the hedged item adjusts the carrying
amount of the hedged item and is recognised in profit or loss. However, if the hedged item is an equity
instrument at FVTOCI, those amounts remain in OCI. When a hedged item is an unrecognised firm
commitment the cumulative hedging gain or loss is recognised as an asset or a liability with a corresponding
gain or loss recognised in profit or loss. [IFRS 9 paragraph 6.5.8]
If the hedged item is a debt instrument measured at amortised cost or FVTOCI any hedge adjustment is
amortised to profit or loss based on a recalculated effective interest rate. Amortisation may begin as soon as
an adjustment exists and shall begin no later than when the hedged item ceases to be adjusted for hedging
gains and losses. [IFRS 9 paragraph 6.5.10]
Cash flow hedge: a hedge of the exposure to variability in cash flows that is attributable to a particular risk
associated with all, or a component of, a recognised asset or liability (such as all or some future interest
payments on variable-rate debt) or a highly probable forecast transaction, and could affect profit or loss. [IFRS
9 paragraph 6.5.2(b)]
For a cash flow hedge the cash flow hedge reserve in equity is adjusted to the lower of the following (in
absolute amounts):
 the cumulative gain or loss on the hedging instrument from inception of the hedge; and
 the cumulative change in fair value of the hedged item from inception of the hedge.
The portion of the gain or loss on the hedging instrument that is determined to be an effective hedge is
recognised in OCI and any remaining gain or loss is hedge ineffectiveness that is recognised in profit or loss.
If a hedged forecast transaction subsequently results in the recognition of a non-financial item or becomes a
firm commitment for which fair value hedge accounting is applied, the amount that has been accumulated in
the cash flow hedge reserve is removed and included directly in the initial cost or other carrying amount of
the asset or the liability. In other cases the amount that has been accumulated in the cash flow hedge reserve
is reclassified to profit or loss in the same period(s) as the hedged cash flows affect profit or loss. [IFRS 9
paragraph 6.5.11]
When an entity discontinues hedge accounting for a cash flow hedge, if the hedged future cash flows are still
expected to occur, the amount that has been accumulated in the cash flow hedge reserve remains there until
the future cash flows occur; if the hedged future cash flows are no longer expected to occur, that amount is
immediately reclassified to profit or loss [IFRS 9 paragraph 6.5.12]
A hedge of the foreign currency risk of a firm commitment may be accounted for as a fair value hedge or a
cash flow hedge. [IFRS 9 paragraph 6.5.4]
Hedge of a net investment in a foreign operation (as defined in IAS 21), including a hedge of a monetary
item that is accounted for as part of the net investment, is accounted for similarly to cash flow hedges:
 the portion of the gain or loss on the hedging instrument that is determined to be an effective hedge is
recognised in OCI; and
 the ineffective portion is recognised in profit or loss. [IFRS 9 paragraph 6.5.13]
The cumulative gain or loss on the hedging instrument relating to the effective portion of the hedge is
reclassified to profit or loss on the disposal or partial disposal of the foreign operation. [IFRS 9 paragraph
6.5.14]
Hedge effectiveness requirements
In order to qualify for hedge accounting, the hedge relationship must meet the following effectiveness criteria
at the beginning of each hedged period:
 there is an economic relationship between the hedged item and the hedging instrument;
 the effect of credit risk does not dominate the value changes that result from that economic relationship;
and
 the hedge ratio of the hedging relationship is the same as that actually used in the economic hedge
[IFRS 9 paragraph 6.4.1(c)]
Rebalancing and discontinuation
If a hedging relationship ceases to meet the hedge effectiveness requirement relating to the hedge ratio but the
risk management objective for that designated hedging relationship remains the same, an entity adjusts the
hedge ratio of the hedging relationship (i.e. rebalances the hedge) so that it meets the qualifying criteria again.
[IFRS 9 paragraph 6.5.5]
An entity discontinues hedge accounting prospectively only when the hedging relationship (or a part of a
hedging relationship) ceases to meet the qualifying criteria (after any rebalancing). This includes instances
when the hedging instrument expires or is sold, terminated or exercised. Discontinuing hedge accounting can
either affect a hedging relationship in its entirety or only a part of it (in which case hedge accounting continues
for the remainder of the hedging relationship). [IFRS 9 paragraph 6.5.6]
Time value of options
When an entity separates the intrinsic value and time value of an option contract and designates as the hedging
instrument only the change in intrinsic value of the option, it recognises some or all of the change in the time
value in OCI which is later removed or reclassified from equity as a single amount or on an amortised basis
(depending on the nature of the hedged item) and ultimately recognised in profit or loss. [IFRS 9 paragraph
6.5.15] This reduces profit or loss volatility compared to recognising the change in value of time value directly
in profit or loss.
Forward points and foreign currency basis spreads
When an entity separates the forward points and the spot element of a forward contract and designates as the
hedging instrument only the change in the value of the spot element, or when an entity excludes the foreign
currency basis spread from a hedge the entity may recognise the change in value of the excluded portion in
OCI to be later removed or reclassified from equity as a single amount or on an amortised basis (depending
on the nature of the hedged item) and ultimately recognised in profit or loss. [IFRS 9 paragraph 6.5.16] This
reduces profit or loss volatility compared to recognising the change in value of forward points or currency
basis spreads directly in profit or loss.
Credit exposures designated at FVTPL
If an entity uses a credit derivative measured at FVTPL to manage the credit risk of a financial instrument
(credit exposure) it may designate all or a proportion of that financial instrument as measured at FVTPL if:
 the name of the credit exposure matches the reference entity of the credit derivative (‘name matching’);
and
 the seniority of the financial instrument matches that of the instruments that can be delivered in
accordance with the credit derivative.
An entity may make this designation irrespective of whether the financial instrument that is managed for credit
risk is within the scope of IFRS 9 (for example, it can apply to loan commitments that are outside the scope
of IFRS 9). The entity may designate that financial instrument at, or subsequent to, initial recognition, or while
it is unrecognised and shall document the designation concurrently. [IFRS 9 paragraph 6.7.1]
If designated after initial recognition, any difference in the previous carrying amount and fair value is
recognised immediately in profit or loss [IFRS 9 paragraph 6.7.2]
An entity discontinues measuring the financial instrument that gave rise to the credit risk at FVTPL if the
qualifying criteria are no longer met and the instrument is not otherwise required to be measured at FVTPL.
The fair value at discontinuation becomes its new carrying amount. [IFRS 9 paragraphs 6.7.3 and 6.7.4]

17.1
1. On January 1, 2011, Doodles Company borrowed P5,000,000 from a bank at a variable rate of interest
for 4 years. Interest will be paid annually to the bank on December 31 and the principal is due on
December 31, 2014. Under the agreement, the market rate of interest on each January 1 resets the
variable rate for that period and the amount of interest to be paid on December 31.
To protect itself from fluctuations in interest rates, the entity hedges the variable interest by entering
into a four-year "receive variable, pay fixed" interest rate swap with a speculator. The interest rate
swap is based on the notional amount of P5,000,000 and an 8% fixed interest rate. The entity has
designated this interest rates swap as a cash flow hedge of the variability of interest payments on the
variable rate loan. Assume that market interest rates are 8% on January 1, 2011, 10% on January 1,
2012, and 11% on January 1, 2013. (Round off present value factors to four decimal places)
The amount to be recognized as derivative asset on December 31, 2012 is
2. On December 12, 2012, Slow Corp. entered into two forward exchange contracts, each to purchase
100,000 euros in ninety days. The relevant exchange rates are as follows:
Forward Rates for
Date Spot Rates March 12, 2013
11/30/12 P87 P89
12/12/12 88 90
12/31/12 92 93
Slow entered into a second forward contract to hedge a commitment to purchase equipment being
manufactured to Slow’s specifications. The expected delivery date is March 2013 at which time
settlement is due to the manufacturer. The hedge qualifies as a fair value hedge. At December 31,
2012, what amount of foreign currency transaction gain from this forward contract should Slow include
in net income?
3. Slow entered into a second forward contract to hedge a commitment to purchase equipment being
manufactured to Slow’s specifications. The expected delivery date is March 2013 at which time
settlement is due to the manufacturer. The hedge qualifies as a fair value hedge. At December 31,
2012, what amount of foreign currency transaction gain from this forward contract should Slow include
in net income?
4. Slow entered into a third forward contract for speculation. At December 31, 2012, what amount of
foreign currency transaction gain from this forward contract should Slow include in net income?
5. On January 2, 2012, Jones Company purchases a call option for P300 on Merchant ordinary shares.
The call option gives Jones the option to buy 1,000 shares of Merchant at a strike price of P50 per
share. The market price of a Merchant share is P50 on January 2, 2012 (the intrinsic value is therefore
P0). On March 31, 2012, the market price for Merchant share is P53 per share, and the time value of
the option is P200. What was the effect on profit of entering into the derivative transaction for the
period January 2 to March 31, 2012?

Determine whether these statements relate to you. Answer Yes No


honestly to check your progress.
I am able to define derivatives and related terms.
I can differentiate derivatives from other types of investments.
I understand the measuring standard for derivatives.
I can determine when to derecognize derivatives from the books.
I can determine the presentation and disclosure requirements for
derivatives.
Total (Raw Score)

You might also like